Spørsmål om 1/x

Her kan du stille spørsmål vedrørende problemer og oppgaver i matematikk for videregående skole og oppover på høyskolenivå. Alle som føler trangen er velkommen til å svare.

Moderatorer: Aleks855, Gustav, Nebuchadnezzar, Janhaa, DennisChristensen, Emilga

Svar
espen180
Gauss
Gauss
Innlegg: 2578
Registrert: 03/03-2008 15:07
Sted: Trondheim

Jeg tenkte at siden [tex]f(x) \to \infty[/tex] når [tex]x \to 0[/tex], så må [tex]\int_{0}^{1} \frac{1}{x}\rm{d}x=\infty[/tex].

Stemmer dette, eller surrer jeg bare?
Vektormannen
Euler
Euler
Innlegg: 5889
Registrert: 26/09-2007 19:35
Sted: Trondheim
Kontakt:

Det bestemte integralet er vel udefinert / eksisterer ikke, siden [tex]\int \frac{1}{x} dx = \ln|x| + C[/tex], og [tex]\ln(x)[/tex] er jo ikke definert for x = 0.
Elektronikk @ NTNU | nesizer
Markonan
Euclid
Euclid
Innlegg: 2136
Registrert: 24/11-2006 19:26
Sted: Oslo

Dette blir et uegentlig integral, ca slik at
[tex]\int_{\small 0}^{\small 1} \frac{1}{x}dx = \lim_{\small s \rightarrow 0}[\ln(x)]_{s}^{1} \;=\; 0 - (-\infty) \;=\; \infty[/tex]

fordi ln(s) -> minus uendelig når s går mot 0 (bare veldig sakte).

Edit
Var riktig ja. 8-)
An ant on the move does more than a dozing ox.
Lao Tzu
Cauchy
Guru
Guru
Innlegg: 359
Registrert: 20/01-2005 11:22

Men argumentet til espen180 holder ikke. Noen funksjoner er svakt singulære, dvs selv om du får et uekte integral eksisterer en endelig grense. Prøv å integrere [tex]\ln{x}[/tex] og så på grensen når x går mot 0 av svaret.
Charlatan
Guru
Guru
Innlegg: 2499
Registrert: 25/02-2007 17:19

Se på symmetrien, vi vet at [tex]1+\int ^{\infty}_{1} \frac{1}{x} \rm{d}x[/tex] er divergent, følgelig må også [tex]\int ^{1}_{0} \frac{1}{x} \rm{d}x[/tex] være det.
Svar